« first day (12 days earlier)      last day (562 days later) » 
00:00 - 20:0020:00 - 00:00

12:13 AM
0
Q: Compute $E(Y|X=m)$

Alex ChavezThe random vector $(X,Y)$ has the following joint distribution $P(X=m, Y=n)$ = $m\choose n$$\frac{1}{2^m}\frac{m}{15}$ where $m=1,..,5$ and $n=0,...,m$. Compute $E(Y|X=m)$

Short question.
 
12:36 AM
0
Q: Is $2sin^2(2x)$ the same as $(2sin(2x))*(2sin(2x))$?

Andre PittrovskyIt is unclear to me how I can manipulate an expression like $2sin^2(2x)$ using identities.

Short question.
 
1:00 AM
0
Q: Lebesgue integrability of step functions

user262860A step function can be defined to be a linear combination of a sequence of brick functions. My question is - Are step functions always Lebesgue integrable ?

Short question.
0
Q: Question related to Gauss-Lucas theorem

Ante PaladinThe (probably) famous Gauss-Lucas theorem states that the roots of the derivative $P'(z)$ are contained in the convex hull of the roots of $P(z)$, where $P(z)$ is complex variable polynomial. I am interested here in could it be the case that we always have some polynomial of any degree (except $...

Words such as question do not add information to titles. Please edit the title so that it better describes the specifics of your question. Do not hesitate to make it longer or include a formula if needed. More tips at: How to ask a good question? (autocomment)Normal Human 21 secs ago
 
0
Q: Is there any way to reject an edit after the fact?

David MolesI ran across a clearly invalid, but approved edit-as-comment by an anonymous user. I can roll it back, obviously; and, given the edit was anonymous, it's not like I could give the original editor-commenter useful rejection feedback in any case. But it feels like there ought to be something strong...

 
0
Q: Is this generalized factorization of a difference of powers correct?

Lebron JamesIs this factorization true? $$(x^n - y^n) = (x+y)^{n-1}(x-y)^{n-1}$$ I am trying to use it in my computation of the determinant of a Vandermonde matrix. Thanks,

Short question.
0
Q: Which family of function does a N/log(N) function belong?

bawejakunalI am not a mathematics wizard. Like a N.log(N) function belongs to quasilinear family of functions, what is the function family that N/log(N) function belongs to ?

Short question.
0
Q: Dividing 5 yards into 50 equal pieces

CamHave 5 yards of ribbon; need 50 equal pieces. 180 inches in 5 yards. So closes I get is 50 pieces...3 inches each ?

Short question.
0
Q: Why? Why? HWo? HWat? HWen? Hwere????????

vroom vroomJust... hwy? Why is that the case? Why? What is the answer? I have no idea. I don't know hwy there are quality Czechs on the problem. 85 down vote accepted Why am I getting this message? All new questions are subjected to a "minimum quality" filter that checks for characteristics of extremely ...

Welcome to Math.SE, vroom vroom. This site uses MathJax formatting of formulas. Tag (soft-question) should not be the only tag a question has. Please add a tag for a subject area to which the question belongs. More tips at: How to ask a good question? (autocomment)Normal Human 21 secs ago
0
Q: Group of order 42 cyclic

user92821As a follow-up from this question: Groups of order 42 I realize that this would probably mean that $G$ is cyclic, could someone walk me through the logic behind this?

Short question.
 
1:39 AM
0
Q: OP doesn't mark question as answered

charlestoncrabbI realize it is the poster's prerogative whether or not to mark a question as answered, but what of the situation in where the OP acknowledges an answer as correct in the comments but does not mark the post as "answered"? More than once I have even been thanked for my response in the comments, bu...

 
0
Q: $\int^0_{-\infty}Ze^{2z}dz$ I am Stuck

CetshwayoI am really trying to solve this problem but I am not following the the solution book. I have used integration by parts. Yet at that point I become stuck. Help me understand. Is the considered a constant? Can I move it outside the indefinite integral? Am I on the right track? It seems as if I a...

0
Q: Quotient space proof

brkLet Y be a supspace of a vector space X. Let x=a+Y=b+Y and y=c+Y=d+Y. How can I show that a+c+Y=a+d+Y=b+c+Y=b+d+Y. I have tried somethings but I couldn't. How should I approach to this proof. Thank you..

Words such as stuck do not add information to titles. Please edit the title so that it better describes the specifics of your question. Do not hesitate to make it longer or include a formula if needed. More tips at: How to ask a good question? (autocomment)Normal Human 21 secs ago
0
Q: A group G with orden 15 is simple?

Mario Alejandro MartinezA group G with orden 15 is simple? There are a theorem for realize it? Thanks for all you help!

Short question.
0
Q: What primary areas of research are there at the intersection of algebra and geometry/topology?

user283871Are there any major areas of modern research that deals with the interactions of algebra and geometry/topology besides the obvious ones such as algebraic topology and algebraic geometry?

Short question.
 
1:57 AM
0
Q: What do I do about a question that belongs on Ask Different?

Caleb KleveterI came across this question that I believe belongs on Ask Different. I was going to flag it as not belonging on this site, but I couldn't: First, does it belong on Ask Different, and if it does, what should I do?

 
0
Q: Implicit Differentiation with partial derivatives.

DiiiGiven that: xs^2+yt^2=1 and x^2s+y^2t=xy-4 where x=x(s,t) and y=y(s,t) find ds/dx, ds/dy, dt/dx and dt/dy. (where these are all partial derivatives) at the point (x,y,s,t)=(1,-3,2,-1)

Welcome to Math.SE, Diii. This site uses MathJax formatting of formulas. More tips at: How to ask a good question? (autocomment)Normal Human 20 secs ago
0
Q: Need help with finding Global extreme values of a function

user2411824I have this function f(x,y)=5x^3-7y , 0<=x , y<=1 Find the values of the global min and max So, I got partial derivatives fx=15x^2 , and fy=-7 But then I run into confusion on how to proceed. Thanks for help

Words such as help do not add information to titles. Please edit the title so that it better describes the specifics of your question. Do not hesitate to make it longer or include a formula if needed. More tips at: How to ask a good question? (autocomment)Normal Human 21 secs ago
0
Q: Differentiation by complex matrix or a numbr

Olivia Cristiana How can I differentiate a complex number by a complex number? How can I differentiate a complex matrix by a complex matrix?

Short question.
 
2:17 AM
2
Q: Is a post such like 'assdddsssafffwq' spam?

Kevin GuanFor example, this post: I think posts like this should be deleted immediately, these posts don't help anybody. So the question is, is that spam? I did flag a post like that spam, now here is that flag: Okay, disputed. So if it's not spam, what can I do? A mod flag? Or just vote to close ...

 
0
Q: Is the function $f(x,y) = \frac{x^2 - y^2}{(x^2+y^2)^2}$ lebesgue integrable in [0,1]^2?

Just4funSo, there's something that I can use to show that $f(x,y) = \frac{x^2 - y^2}{(x^2+y^2)^2}$ is or is not Lebesgue integrable in [0,1]^2?

Short question.
0
Q: On chinese remainder theorem

ArulGiven $a\equiv x \bmod r_1$ and $b\equiv x\bmod r_2$, we can construct $x$ from $$x=a r_2 [r_2^{-1}]_{r_1} + b r_1 [r_1^{-1}]_{r_2}$$ where $(r_1,r_2)=1$. Suppose $(r_1,r_2)=r$ and $a\equiv x \bmod r_1$ and $b\equiv x\bmod r_2$ and $a\equiv b \bmod r$ holds then what is the constructive formula ...

Questions tend to get more attention when they have a tag for a broad area of mathematics relevant to the question. Some of these tags might fit. (autocomment)Normal Human 20 secs ago
0
Q: Word problem help, Design a rectangular milk carton box

user2411824This is the problem: Design a rectangular milk carton box of width w, length l, and height h which holds 520 cm^3 of milk. The sides of the box cost 2 cent/cm^2 and the top and bottom cost 3 cent/cm^2. Find the dimensions of the box that minimize the total cost of materials used. My approach was...

Words such as help do not add information to titles. Please edit the title so that it better describes the specifics of your question. Do not hesitate to make it longer or include a formula if needed. This site uses MathJax formatting of formulas. More tips at: How to ask a good question? (autocomment)Normal Human 21 secs ago
 
2:35 AM
0
Q: Can we stop pestering new users?

R MenkeI am a Swift developer, which is a hyped language and most of the questions are from very new users on SO. Almost every first question asked by a new user is bad. It's something we all went through. Swift/iOS has lot's of them. New users just don't understand yet that the voting system is not ...

 
0
Q: Proving that if $f(x) \le g(x)$ then $f(x_0) \le g(x_0)$

Ryan T. DonnellySuppose $f: D \to R$ and $g:D \to R$, $x_0$ is an accumulation point of $D$, and $f$ and $g$ have limits at $x_0$. If $f(x) \le g(x)$ for all $x \in D$, then $\lim \limits_{x \to x_0}f(x) \le \lim \limits_{x \to x_0}g(x)$ Assume that $\forall x \in D$, $f(x)$ and $g(x)$ both have limits at s...

Tagged proof-verification.
 
3:01 AM
0
Q: Subgroups of index $p$ in additive group $\mathbb{Z}^2$?

Michael SmithHow many subgroups of index $p$, where $p$ is a prime, are there in the additive group $\mathbb{Z}^2$?

Short question.
0
Q: Can a hyperbolic surface be isometrically embedded into R^4?

Hyperbolic AskerCan a complete hyperbolic surface be isometrically embedded into flat R^4?

Short question.
0
Q: Solutions of Sturm-Liouville Eigenproblem Dependent on Eigenvalue as Parameter

VladLet $\;y_1\left(x,\,\lambda\right),\: y_2\left(x,\,\lambda\right)\;$ be respective solutions of eigenvalue problem \begin{align}y\,'' &= \lambda \,y, & x&\in\big(\,0,\,\infty\,\big)\end{align} with boundary conditions \begin{align} &\begin{cases} y_{1}\big(0,\,\lambda\big)=1\\y_{1}'\big(0,\,\lamb...

Consider replacing (analysis) with a more specific tag for the relevant branch of analysis. (autocomment)Normal Human 21 secs ago
 
3:21 AM
-1
Q: Why is my account renewed

Mini CascadingIt has been little since I visited my SO account my earlier account was when I wanted to Login I faced a scenario as I'm new user, system was asking permissions and so on and so forth. I was a user with 15+ reputation but when I logged in got 1 reputation on my account looks like as I'm new S...

 
0
Q: Computing a sum of functions of $\xi$-adhering numbers: $\big\lfloor{\xi \over\lfloor{\xi/\beta}\rfloor}\big\rfloor=\beta$

KugelblitzAssume the existence of a $\xi \in Z^+$. A $\beta \in Z^+ ; 1\le\beta\le \xi$ is said to be $\xi$-adhering if $$\big\lfloor{\xi \over\lfloor{\xi/\beta}\rfloor}\big\rfloor=\beta$$ Let $f(\xi)$ be the number of $\xi$-adhering numbers for each integer $\xi\ge 1$. Question: Compute $f(1)+f(2) \c...

Tagged proof-writing.
 
3:39 AM
0
Q: How to find the rate at which something is increasing?

ChrisCertain pieces of antique furniture increased very rapidly in price in the 1970s and 1980s. For example, the value of a particular rocking chair is well approximated by V = 105 (1.75)^t, where V is in dollars and t is the number of years since 1975. Find the rate, in dollars per year, at which...

0
Q: Proving using assumptions

P999Given the assumption, I am asked to prove the property of floor and ceiling functions. Is my approach correct for using that assumption by showing that the assumption's pre-conditions are valid and the rest?

Tagged proof-verification.
0
Q: Algebra logic on simplification

JosephI've been self studying algebra for some time alone and a common problem I bump into that always throws me off is something like this (Complete idiots guide to algebra, if you know a better book please tell.) simplify 5-(-3)-(+2)+(-7) Now the double signs are removed so the non brackets become ...

Welcome to Math.SE, Joseph. This site uses MathJax formatting of formulas. More tips at: How to ask a good question? (autocomment)Normal Human 21 secs ago
0
Q: Geometric progression with negative and possitive exponentiation

tomiHow this sum can be performed if not with the use of usual formula? (1/6)(SUM from -2 to n)(0.5*e^(-jpik/3))^n

0
Q: A cat chases a rat. For every 5 leaps of the rat , the cat takes 3 leaps.

rstproblem A cat chases a rat. For every $5$ leaps of the rat , the cat takes $3$ leaps, but the $2$ leaps of the cat are the same as $3$ leaps of the rat. Compare the speeds of the cat and the rat a)$4:5$ b)$12:13$ c)$16:15$ d)none of these

 
3:55 AM
0
Q: Disciplinary action for nasty behaviour

P VanchinathanA couple of hours ago I reviewed a proposal to edit a post. The suggested edit was nasty, irrelevant and of pornographic nature. I rejected the edit choosing the reason for rejection as "harmful". I added a comment saying I recommend the user who suggested the edit be expelled. Will this get es...

 
Consider adding a tag for a broader subject area to which the question belongs. Some of these tags might fit. (autocomment)Normal Human 21 secs ago
0
Q: Statistics : Permutations and Combinations

Abdullah ZameekIn how many ways can 2 different prizes be given to 10 students if one student may receive both prizes?

Short question.
0
Q: How to become CEO of Walt Disney Corporation?

Charge of CoatsDon't get me wrong I don't really want to but I need to in order to have experience. I want Thomas Staggs position as chief executive officer of the entire corporation. Thanks

Short question.
0
Q: Use Cauchy-Schwarz Inequality to prove statement

RatonStubleyUse the Cauchy-Schwarz inequality to show that (acos(θ)+bsin(θ))$^2$ $\leq$ a$^2$ +b$^2$ for all a,b,θ ∈ $\mathbb{R}$ What I was trying to do was to take the smaller of either a or b and prove that (acos(θ)+bsin(θ))$^2$ $\leq$ a$^2$ if a were smaller than b. Then I created two vectors $\vec{v}$,...

Tagged proof-writing.
0
Q: Problem dealing with $\sum \frac{\sin(n)}{n}$ and its convergence

rashid $$\text{If} \ S=\displaystyle\sum_{n=1}^{\infty}\dfrac{\sin (n)}{n}, \ \text{then what is} \ 2S+1$$ I know that $\sum \frac{\sin(n)}{n}$ converges. But now what do I do?

Title contains problem.
 
4:31 AM
0
Q: Why are yes/no type questions discouraged?

sashaI have seen experienced users and moderators comment on some questions that cs stackexchange does not support yes/no type questions ( not the exact words ). But if the OP discusses his/her approach and is not able to prove it thoroughly and asks whether what he/she is trying to prove if correct o...

 
4:41 AM
0
Q: Convolution cannot p-p operator unless p=2

Sayan Let $g=e^{-ix^2}, x\in \mathbb{R}$. Let $T$ be an operator defined as $T(f)=f*g$. Show that $T$ cannot satisfies $p-p$ inequality unless $p=2$. Note: We say an operator $T$ satisfies $p-p$ inequality if $||Tf||_p\le C||f||_p$ whenever the expression on the right side exists finitely. An...

Title contains 2.
0
Q: Writing a doubling equation given only amount of time to double.

David ScidmoreWrite an equation to model the generation of Ecoli if the doubling time is 20 minutes. I can't for the life of me figure this out.

Short question.
 
4:54 AM
0
Q: Quadratic residue basic property

Mualpha7Prove that if $a$ is a quadratic residue modulo $p$ and modulo $q$, then $a$ is a quadratic residue modulo $pq$.

Consider adding a tag for a broader subject area to which the question belongs. Some of these tags might fit. (autocomment)Normal Human 21 secs ago
0
Q: to calculate 1 side measurement as I had 3 side perimeter measurements & the total land area.

K S KuehThe total area is 4046 sq meter. The 3 side perimeter are 126.636 meter, 31.762 meter & 127.275 meter. What is the length of the 4th side?

Welcome to Math.SE, K S Kueh. Consider adding a tag for a broader subject area to which the question belongs. Some of these tags might fit. (autocomment)Normal Human 20 secs ago
0
Q: Find the equations of the locus of the point P (x,y) that is equidistant from the lines 4x+3y-2=0 and 12x-5y+6=0.

KateFind the equations of the locus of the point P (x,y) that is equidistant from the lines 4x+3y-2=0 and 12x-5y+6=0. Do I use simultaneous equations? I cant remember:(

Welcome to Math.SE, Kate. Questions tend to get more attention when they have a tag for a broad area of mathematics relevant to the question. Some of these tags might fit. (autocomment)Normal Human 20 secs ago
0
Q: Proof that any a >= 20 Can be Written as a = 5b + 6c Where a, b and c are Natural Numbers

VortronI'm currently working on a problem for an assignment, so forgive me if my question is a little clumsy or vague. I'm trying to get myself headed in the right direction and offer a meaningful question here without just being handed the answer. I'm trying to prove that $\forall$$a$ $\epsilon$ $\mat...

Welcome to Math.SE, Vortron. Tag (proof-explanation) should not be the only tag a question has. Please add a tag for a subject area to which the question belongs. (autocomment)Normal Human 20 secs ago
0
Q: A summation series of binomial coefficients

user18Given. (1 + x^2005 + x^2006 + x^2007)^2008 = A0 +A1*X +A2*X^2 + .... An*X^n We are required to calculate A0 - A1/3-A2/3 + A3 -A4/4 -A5/5 + A6 ...... I tried approaching the problem by setting x to 1 and ω in two different cases. For x=1, we get sum of all coefficients as 4^2008 For x=ω , w...

Welcome to Math.SE, user18. This site uses MathJax formatting of formulas. More tips at: How to ask a good question? (autocomment)Normal Human 20 secs ago
0
Q: Coordinates change and fubini theorem

user289920i wanted just to understantd how this equality is found, by change of cordinnates \int_{0}^{1}{1/{x^{p-1}}\int_{0}^{1/x}|ln{t}|^p/|1-t|^p dtdx= \int_{0}^{1}|ln{t}|^p/|1-t|^p \int_{0}^{1}{1/{x^{p-1}}dxdt + \int_{0}^{+\infty}|ln{t}|^p/|1-t|^p \int_{0}^{1/t}{1/{x^{p-1}}dxdt thanks

Welcome to Math.SE, user289920. This site uses MathJax formatting of formulas. More tips at: How to ask a good question? (autocomment)Normal Human 20 secs ago
0
Q: What is the value of $f(1+i)$??

KprimeXIt is given that $\text{Im}(f'(z))=6x(2y-1)$ and $f(0)=3-2i$, $f(1)=6-5i$ then what is the value of $f(1+i)=?$. I tried by assuming $f(z)=u+iv$ and $z=x+iy$ so $\text{Im}f'(z)=v'=6x(2y-1)$ but couldn't reach to a profitable conclusion. Any help or hint please.

Title contains ??.
0
Q: Question about normality of translations in Affine group.

user139985 So I understand the construction of this semi-direct product and it's properties. However I am confused to why $K^n$ is normal in Aff(n,K). My main confusion comes from this. I believe understand how T$_{A,b}$ $\in$Aff(n,K) acts on x$\in$K$^n$. It would be T$_{A,b}$(x)=Ax+b right? But how d...

0
Q: Geometry question on Incentre

AyushLet ABC be a triangle in which AB = AC and let I be its in-centre. Suppose BC = AB + AI. Find ∠BAC.

Words such as question do not add information to titles. Please edit the title so that it better describes the specifics of your question. Do not hesitate to make it longer or include a formula if needed. More tips at: How to ask a good question? (autocomment)Normal Human 20 secs ago
0
Q: Exercise 1.2.3 (From Grimmet and Stirzaeker)

QuasarI am new to axiomatic probability and am self-learning this topic. This question is from Grimmet's book. A conventional knock-out tournament(such as that at the Wimbledon) begins with >$2^n$ competitors and has $n$ rounds. There are no play-offs for the positions >$2,3,...,{2^n}-1$, and the ...

Title contains exercise.
 
5:49 AM
0
Q: I have this question from higher algebra by Hall and Knight

Sanjeev VermaI have this question from higher algebra by Hall and Knight: if $ \frac{y}{x-z} = \frac{x+y}{z} = \frac{x}{y} $ then find the ratio of x,y and z? There are two answers given for this question , the first is $\frac x4 =\frac y2 =\frac z3$ and the second is $\frac x1 =\frac y{-1} =\frac z0 $.Now i...

Words such as question do not add information to titles. Please edit the title so that it better describes the specifics of your question. Do not hesitate to make it longer or include a formula if needed. Questions tend to get more attention when they have a tag for a broad area of mathematics relevant to the question. Some of these tags might fit. More tips at: How to ask a good question? (autocomment)Normal Human 21 secs ago
0
Q: {(1,3),(1,4),(2,3),(2,4),(3,1),(3,4)}, this set had no relation. Is it true and why?

user3572326{(1,3),(1,4),(2,3),(2,4),(3,1),(3,4)} this set had no relation. Is it true and why?

Short question.
 
6:07 AM
0
Q: Exercise 1.2.4 (From Grimmet and Stirzaeker)

QuasarThis is yet another problem, where I have run into trouble. Let ${F}$ be a ${\sigma}$-field of the subsets of $\Omega$ and suppose that $B\in{F}$. Show that $G=\{A\cap{B}:A\in{F}\}$ is a $\sigma$-field of the subsets of $B$. I know a $\sigma$-field is, and that it is closed under complement...

Title contains exercise.
0
Q: How do I evaluate this summation?

Anders GustafsonI was wondering how do I solve the summation? I found it in this link on power towers http://mathworld.wolfram.com/PowerTower.html

Short question.
0
Q: How to solve simultaneous equations of exponential powers?

edificationHow to solve the following simultaneous equations? x + y + z = 0 x^2 +y^2 +z^2 = 1 x^3 +y^3 +z^3 = 0 Thank you!

Welcome to Math.SE, edification. This site uses MathJax formatting of formulas. More tips at: How to ask a good question? (autocomment)Normal Human 21 secs ago
0
Q: Basic intersection/union of sets

Jon Mark PerryThis might sound like an easy question, but I need to prove: $A$ is a set. $A\cap A=A$ $A\cup A=A$ $A\setminus A=\emptyset$ I have tried; let $x\in A \therefore x\in A$.

Title contains basic.
0
Q: Convergence in norm

user262860If {$f_k $} is a sequence of Lebesgue integrable functions, then {$f_k$} is said to "converge in norm" to an integrable function $f$ if $\int | f_k - f | $ converges to zero . Can someone explain to me the concept behind convergence in norm ?

Consider replacing (analysis) with a more specific tag for the relevant branch of analysis. (autocomment)Normal Human 21 secs ago
0
Q: A problem on left Fredholm Operator..

Black -horseI was reading Fredholm Operators from the book "A course in Functinal Analysis " by J.B Conway. There I got stuck with the following problem. Let $A\in B(\mathcal H)$. Show that $A(\mathcal M)$ is closed for every closed subspace $\mathcal M$ of $\mathcal H$ iff $A$ has finite rank or $A$ is le...

Title contains problem.
0
Q: Proving orthogonal vectors

RlocShow that two nonzero vectors $\vec{v_1}$,$\vec{v_2}$ ∈ $\mathbb{R_3}$ are orthogonal if and only if their direction angles satisfy cos$α_1$ cos$α_2$ +cos$β_1$ cos$β_2$ +cos$γ_1$ cos$γ_2$ =0. Note: I tried to turn all of the cos$[angle]_2$ to sin[angle] and then convert all of the cos[angle] ter...

Tagged proof-writing.
 
6:38 AM
0
Q: Find The value of $c$ at given points

Syed Muhammad AsadWe have equation $$c=\frac{y-1}{xy}$$ Find the value of c using initial value condition $y(0)=1$ i.e. when $x=0 \implies y=1$

Short question.
0
Q: question on factors of numbers

Sarnavo SarkarLet N be a positive integer And N=$(a^p)(b^q)$ Then prove that sum of the divisors or factor of N is =$$({a^p+1}-1)({b^q-1}-1)/(a-1)(b-1)$$

Words such as question do not add information to titles. Please edit the title so that it better describes the specifics of your question. Do not hesitate to make it longer or include a formula if needed. More tips at: How to ask a good question? (autocomment)Normal Human 21 secs ago
0
Q: Existence of positive integers a1,a2,...,ak

Turtle MathLet x and y be positive integers such that arctan(1/x)+arctan(1/y)<(pi/2). Show that there exists positive integers a1,a2,...,ak none of which equals x or y such that arctan(1/x)+arctan(1/y)+arctan(1/a1)+arctan(1/a2)+...+arctan(1/ak)=(pi/2). I simplified the given inequality to arctan((1/x+1/y...

Welcome to Math.SE, Turtle Math. This site uses MathJax formatting of formulas. More tips at: How to ask a good question? (autocomment)Normal Human 21 secs ago
0
Q: Perpendicular (orthogonal) vector proof

RlocShow that two nonzero vectors $\vec{v_1}$,$\vec{v_2}$ ∈ $\mathbb{R_3}$ are orthogonal if and only if their direction angles satisfy cos$α_1$ cos$α_2$ +cos$β_1$ cos$β_2$ +cos$γ_1$ cos$γ_2$ =0. Note: I tried to turn all of the cos$[angle]_2$ to sin[angle] and then convert all of the cos[angle] ter...

Tagged proof-writing.
0
Q: irreducibility of a polynomial over $\mathbb{F}_2$

irrIs there any criterion (theorem or algorithm) to find out a given polynomial $f(x)$ is irreducible over the field $\mathbb{F}_2$?

Short question.
0
Q: Covariance stationarity of AR(1)-ARCH(1)

A_JoI'm becoming confused by this. Say I have the following model: $$ y_t = c+\phi y_{t-1} +\epsilon_t \,, \epsilon_t|\Omega_{t-1} \tilde{} WN(0,\sigma_t^2 ) $$ $$ \sigma_t^2=\alpha_0+\alpha_1\epsilon_{t-1}^2 $$ $$ |\phi|<1 \,, \alpha_1<1, \alpha_0 \ge 0, \alpha_1>0 \,. $$ I know that an AR(1) i...

Welcome to Math.SE, A_Jo. Questions tend to get more attention when they have a tag for a broad area of mathematics relevant to the question. Some of these tags might fit. (autocomment)Normal Human 21 secs ago
0
Q: Foot of perpendicular on a chord of a conic

Sat DFor a standard ellipse, a chord subtends an angle of 90 degrees with the centre (0,0). To find the locus of the foot of perpendicular to this chord from the centre of the ellipse, I wrote the equation of chord with parameters A, B: xcos(A+B/2)/a + ysin(A+B/2)/b = cos(A-B/2) Since an angle of 90 i...

Short question.
 
7:19 AM
0
Q: Delete undelete or delete cycling to clear flags

user193661The flags on this question dismissed as helpful at the same time OP delete cycled it. Hmmm So, do yem theink that any provisions need for it?

 
7:31 AM
0
Q: Are SO questions by default permitted to evolve? If yes to what extend?

sjsamIt is a usual case that the initial question which is posted may undergo a series of changes during the troubleshooting/debugging process and the final version of the question might not have anything to do with the first version of it. I would call such thing as 'evolution' though I am not sur...

 
0
Q: Fluid mechanics

siddabasappaIs thermal conductivity(thermal conductivity) of nano sized particles and bulk particles are same ? If no whats the difference and is there any relation between property and size ?

Welcome to Math.SE, siddabasappa. Consider adding a tag for a broader subject area to which the question belongs. Some of these tags might fit. (autocomment)Normal Human 20 secs ago
0
Q: If cos x = 3/5 and x lies between pi and 2pi, find the value of tan x.

Archie DeetooI have used the pythagorean identity to show that tan x = 4/3. Then I used the unit circle to see that the angle lies in the third and/or fourth quadrant because they are between pi and 2pi. This means that tan in those quadrants is either positive (in the third quadrant) or negative (in the fou...

Welcome to Math.SE, Archie Deetoo. This site uses MathJax formatting of formulas. More tips at: How to ask a good question? (autocomment)Normal Human 21 secs ago
 
0
Q: What is the advantage of expiring close votes?

celWhy do close votes expire after some time? To me this concept seems to have mostly disadvantages. Just because some time passed, why should the system or I have changed my mind about the off-topicness of a question? I understand that for frequently viewed/voted questions it makes litte sense to ...

 
7:58 AM
0
Q: What is the proof, instance, and verifier mean in the definition of NP problem?

WieshawnI came across a definition of NP problems: Definition. A decision problem $X ∈ NP$, if there exists a polynomial time verifier $V$ such that For every yes instance $x ∈ X$, there exists a polysized proof y such that $V (x, y) = yes$. For every no instance $x ∈ X$, and for every proof y, we ha...

Title contains problem.
0
Q: IS the following complex series convergent or divergent?

ProbabilityGuyI am trying to study the series $$ \sum_{n=1}^{\infty} \frac{ (-1)^k (\log n)^k}{n^2} $$ Where $k \in \mathbb{N}$. What test would work in this case?

Short question.
0
Q: Proof/intuition that any number can be expressed in binary form and every number will have a unique representation?

Aditya AgarwalI was just thinking lately that how do we know that literally every number can be expressed in binary? And that too, with a unique representation?

Short question. Tagged proof-writing.
0
Q: Anyone has a different tally mark sign which represents more number of objects, more comprehensible?

Aditya AgarwalWe all have seen the basic tally mark sign: So anyone has a better notation, which includes more objects in one grouping, more informative and easily readable? (As compared to this)

Consider adding a tag for a broader subject area to which the question belongs. Some of these tags might fit. (autocomment)Normal Human 21 secs ago
0
Q: What is the difference between Bilinear function and Bilinear form?

Vishwas JainI am going through a textbook on Matrix Algebra and I am not able to understand the difference between the two!

Short question.
 
8:23 AM
0
Q: New Users Chat by Invitation?

Darwin von CoraxI've had occasion recently (I've only been here two weeks, but whatever...) to answer a new member's question, only to quickly realize that the individual's misconceptions or lack of knowledge exceed what I feel comfortable covering in answers or comments, and that an extended chat is called for,...

 
Hello
2
Q: How to calculate delta on Android

Polarbear0106How do I calculate the time between frames on Android? I'm using this for the thread: public class GameThread extends Thread { private int FPS = 30; private double averageFPS; private SurfaceHolder surfaceHolder; private Game gamePanel; public boolean running; public ...

 
0
Q: Derivative of a function

student123Which is the derivative of $f: \mathbb{R} \to \mathbb{R}, f(x)=x^c, c \in \mathbb{R}$ ? Is the same as in the case where $ c \in \mathbb{N}$

Short question.
0
Q: An exercise from Lars Ahlfors's complex analysis

Chunqiu SongChapter 4, exercise 2.3, problem 5 I can't understand the first part of the problem, the inequality seems holds for some functions at some points. Is the problem wrong?

Title contains exercise. Short question.
0
Q: Complete and correct deductive axiom for SOL

Raúl Aparicio BustilloDoes exist a complete, correct but no recursive deductive axiom for second order logic?

Short question.
 
8:46 AM
0
Q: How is the Inverse function theorem used to prove that the formulae in this question are the same?

BLAZEI was informed in my last question that the Inverse function theorem: $$(f^{-1})^{\prime}(f(a))=\cfrac{1}{f^{\prime}(a)}\tag{I*}$$ was needed to show that $$\rho_x (x)=\rho_\alpha(\alpha)\left|\frac{\mathrm{d}x}{\mathrm{d}\alpha}\right|^{-1}\tag{A}$$ is the same formula as $$\rho_y(y)=\rh...

Consider adding a tag for a broader subject area to which the question belongs. Some of these tags might fit. (autocomment)Normal Human 21 secs ago
 
9:05 AM
0
Q: Suppose A is a 5 x 8 matrix with rank A = 5

BNSlugjust some basic concept questions to clear up some definitions: Does Ax = 0 have a non-trivial solution? Why? My answer to this is no, because to have a rank 5 that means there is a pivot position in every column and therefore can't have a non trivial solution. Is this right? I feel as though ...

Title contains 5.
 
9:43 AM
0
Q: Why is this function is $O(x^n)$

quinque$$f(x) = \prod_{k=0}^{n} (1+kx)^{ (-1)^k \binom {n} {k} }$$ How to prove that Taylor expansion of this function at zero stars from $x^n$ (all lower terms are zero)?

Short question.
0
Q: Function to repeat number N times

Bob NapkinWhich type of function can I use to repeat one number? For example: number 2 repeat 3 times and get 222?

Tag (contest-math) should not be the only tag a question has. Please add a tag for a subject area to which the question belongs. (autocomment)Normal Human 21 secs ago
 
9:59 AM
0
Q: How to properly suppress irrelevant comments on a question?

nowoxI often ask questions that gets many comments. To increase the readability of the question I usually edit the question and add all the details discussed on the comments. So these comments become useless. What I usually do is write a new comment: @user: I will remove my comments since I update...

1
Q: Why is this answer bad but this one awesome?

T3 H40I Just failed this audit, and I would like to understand why. The answer is for a (9 month old) question with zero vote and low views. Additionally, there is No other answer, so this is definitely no "I want to answer a famous question to get some rep" type of answer. Lets have a look at it: ...

1
Q: How do I search my own answers.

Georges ElencwajgThis morning I wanted to find an answer of mine proving that some plane curve is not hyperelliptic. So I typed not hyperelliptic Georges Elencwajg in the search box and got a message reading "0 results". But when I typed just not hyperelliptic I got 92 results, of which the very first contained ...

0
Q: How should we react to exact copied answers?

Hi I'm frogattoYesterday I came across this answer that is exactly copied and pasted from its upper answer. This is definitely considered as plagiarism. I commented "Please delete your answer.". But she didn't, as another user also already had commented that this is not ethical. In short, my question is: How s...

 
10:46 AM
0
Q: Sweet - Question

Algorithm era'There are n sweets in a bag. Six of the sweets are orange. The rest of the sweets are yellow. 'Hannah takes a sweet from the bag. She eats the sweet. Hannah then takes at random another sweet from the bag. She eats the sweet. 'The probability that Hannah eats two orange sweets is 1/3. Show tha...

Words such as question do not add information to titles. Please edit the title so that it better describes the specifics of your question. Do not hesitate to make it longer or include a formula if needed. More tips at: How to ask a good question? (autocomment)Normal Human 20 secs ago
0
Q: Need help with my double differentiation matrix. Calculations provided

user289419Let S :P3(F)--> P2(F) be diferentiation twice. Then the matrix M(S,(1,x,$x^2$,$x^3$),(1,x,$x^2$) of S with respect to the bases (1,x,$x^2$,$x^3$) and (1,x,$x^2$) of P3(F) and P2(F) respectively is f 'x(1)= 0.1+0.x+0.$x^2$+0.$x^3$ f 'x(x)=1.1+0.x+0.$x^2$+0.$x^3$ f ' x($x^2$)= 0.1+2.x+0.$x^2$+0.$...

Title contains help.
0
Q: f(x,y)=ln(x^2+y^2)/(1-x^2-y^2) uniformly continuous?

Wiktoria CzerwonkaIs f(x,y)=ln(x^2+y^2)/(1-x^2-y^2) uniformly continuous on {(x,y):x^2+y^2<1}: I know the definition of uniform continuity and Since f=g∘r is the composition of UC functions, it is also UC I need a hint, please help.

 
11:12 AM
0
Q: Compute the Fourier Series for $f(x)=cos(\frac{x}{2}) (-\pi<x≤\pi) and $f(x)=f(x+2)

kRiShNa_96I know that b_{n} is an even function, due to the function being an even function, however I am struggling to compute a_{n}

Short question.
0
Q: How to find$\int{x(ln(x))^3}{dx}$ with change of the variable?

Matt Peyvandi$\int{x(ln(x))^3}{dx}$ Hi, I was hoping you could help me solve this with changing the variable. Thanks

Short question.
0
Q: Proving that (n/n^2) converges to 0

Yonatan IzutskiverI'm completely clueless on this one. I can easily calculate the limit using L'Hopital's rule, but proving that the series is converging to 0 is far more tricky. $$a_n= {n \over 2^n}$$ Any help?

Title contains 0.
 
11:31 AM
0
Q: Why is the parameter (t-1) in this example?

LightvvindExample 1 on this page: http://mathinsight.org/parametrized_curve_tangent_line_examples Why do they use $(t-1)$ in the last step ($l(t)=c(1)+(t−1)c′(t_0)$)? Why not just use $t$?

Short question.
0
Q: Why define $(\nabla^2F)(X,Y)=\nabla_X(\nabla_YF)-(\nabla_{\nabla_XY}F)$?

lanse2ptyWhy define $(\nabla^2F)(X,Y)=\nabla_X(\nabla_YF)-(\nabla_{\nabla_XY}F)$? I can't find the motivation of this define.I don't know the purpose of defining so.

Short question.
0
Q: solving lim x->+inf of lan(x^2 - x +1)/lan(x^10 + x +1)

Bak1139hints on solving lim x->+inf of lan(x^2 - x +1)/lan(x^10 + x +1) would be appreciated. i tried multiplying each polynomial with the inverse of highest power but it didnt work out. no lhospital

0
Q: Please help me find the 4th root of -64

Olegas RudgalvisI need to find the 4th root of -64. I have the answers but I don't know how to do it. This has to be done without the use of De Moivre's theorem as we haven't been taught it. Thank you very much for any and all help :)

Words such as please, help do not add information to titles. Please edit the title so that it better describes the specifics of your question. Do not hesitate to make it longer or include a formula if needed. More tips at: How to ask a good question? (autocomment)Normal Human 21 secs ago
0
Q: Range and nullspace of this 4 by 2 matrix, no calculations

user289419Linear map from R2 to R4 1 1 2 2 3 3 4 4 Now I know the null-space is everything in the form (x,-x) so this is one dimensional. Apparently the range is also one dimensional however if I apply my matrix to say (x,y) I get (x+y,2(x+y),3(x+y),4(x+y)) transposed. The answer is that the range is...

0
Q: Treating Derivative as a Function

batmanthethirdI am currently looking through a proof in a book and am failing to understand one particular step. The step is this: $\sum_{i=0}^{\infty} x(\frac{d}{dx})x^n= x(\frac{d}{dx})\sum_{i=0}^{\infty}x^n$ How is it that you can treat the: "$x(\frac{d}{dx})$" term as a constant here? Does the derivative...

Questions tend to get more attention when they have a tag for a broad area of mathematics relevant to the question. Some of these tags might fit. (autocomment)Normal Human 21 secs ago
 
11:59 AM
0
Q: Prove that the fraction(21n+4)/(14n+3)...

Algorithm eraProve that the fraction(21n+4)/(14n+3) is irreducible for every natural number n. I know the solution already. Just for sharing new idea.

Short question.
0
Q: Finding limit of sequence 6

ParketPlease, help me to find limit of this sequence: $\lim_{n\to \infty} \left(\frac{n^2 + n}{n^2 + n + 2}\right)^n$

Title contains 6. Short question.
 
12:16 PM
1
Q: Do I have to award the bounty even if the answer does not deserve it and is downvoted?

AniketDo I have to award the bounty even if the answer does not deserve it and is downvoted? This is in relation to this question of mine where there is only one and that too downvoted answer..

 
0
Q: Inequalities involving Maclaurin Series of 1-e^-x

Shrey AryanSuppose you are given with the first three terms of the Maclaurin Series expansion of the function f(x)=1-e^-x Then show, with reasoning, that f(x)>x-x^2/2 for 0 Now, I began by deriving the expansion and it was like this f(x)=1+x/1-x^2/2+.... But I don't understand the logic behind the i...

 
0
Q: How can bash question is on-topic and C programming off-topic?

edward torvaldsI have read this answer it suggests that Bash questions are on-topic since it helps in administration. Keeping this in mind, C programming questions too should be on-topic. we can use C to write services and routine and therefore helps in administration. To me it sounds like there is no specific...

 
0
Q: $\int_{ln4}^{ln9} e^\frac{x}{2}$ dx = 2?

SoroushI don't know why ? $$\int_{ln4}^{ln9} e^\frac{x}{2} dx = 2$$ i calculated 1.

Short question.
0
Q: Is $ \left| \lim_{t \rightarrow 0}{\dfrac{f(x+ty) - f(x)}{t}} \right| \leq \lim_{t \rightarrow 0}{ \dfrac{|f(x+ty) - f(x)|}{|t|}}$

IdonknowSuppose $X$ is a Banach space and $f:X \rightarrow \mathbb{R}$ is a continuous function. Is it true that $$ \left| \lim_{t \rightarrow 0}{\dfrac{f(x+ty) - f(x)}{t}} \right| \leq \lim_{t \rightarrow 0}{ \dfrac{|f(x+ty) - f(x)|}{|t|}}$$ where $t \in \mathbb{R}, x,y \in X$ ? I think it is true wh...

Tall formulas in titles break the layout of question lists. Please replace \dfrac with \frac in the title. (autocomment)Normal Human 24 secs ago
0
Q: Alternative proof for the irrationality of 2; questions

SelfStudySo I started reading Conjecture and Proof by Miklos Laczkovich and one of the first proofs he provides is that of the irrationality of the square root of two. I am aware there are alternative proofs (one of which is geometric and another that uses the fundamental theorem of arithmetic) but I have...

Tag (proof-explanation) should not be the only tag a question has. Please add a tag for a subject area to which the question belongs. (autocomment)Normal Human 21 secs ago
 
12:43 PM
0
Q: bounty not awarded

BlackbeltI answers this question before the op started the bounty and after couple of edits he accepted the answer. At the end of the grace period the bounty wasn't awarded. is it the expected behavior? IMO the accepted answer should be always automatically be awarded if the bounty wasn't manually awarded...

 
0
Q: Equality of complex numbers in trigonometric form

A6TechHow are these two equal? My teacher said it is obvious, am I missing something? $$\prod_{k=1}^{n}(\cos{kx}+i\sin{kx})=\cos{(1+2+...+n)x}+i\sin{(1+2+...+n)x}$$

Short question.
0
Q: Finding the limit of sequence.

ParketPlease, help to proof that: $$\lim_{n\to \infty} \left(\frac{(\sqrt[n]{a} + \sqrt[n]{b})^n}{2 ^ n}\right) = \sqrt{ab}$$

Short question.
0
Q: What is the distribution of the position of the maximum of a Brownian bridge?

Ben DerrettWhat is the distribution of the position of the maximum of a Brownian bridge?

Short question.
0
Q: Book for Functional Analysis

ketumI need a book or lecture notes for the course functional analysis, which I took this semester, the lecturer mentioned some book at the beginning and currently I'm also reading the book of Vitali Milman but some chapters, we treated in the lecture, are not contained in the book (the starred chapte...

0
Q: Prove the equation lnx=1/x has a unique solution

N. OzturkThe question is like: Prove that the equation ln(x)=1/x for x>0 has a unique solution and explain why. When it asks about the "unique solution" I try to find the exact value. Is it possible to find it, how would I solve it? Thanks.

Tagged proof-verification.
0
Q: the space $\mathcal{C}(\Omega),\quad \mathcal{C}^1(\Omega)$

HB khaledIs the spaces $\mathcal{C}(\Omega),\quad \mathcal{C}^1(\Omega),\;\Omega \;\mbox{open set in} \;\mathbb{R}$ hilbert ? If so, how to prouve that ?

Short question.
 
1:19 PM
-2
Q: Bounty on a question on meta

AniketWhy don't we have the chance to put a bounty on a question on meta? There may be cases where a person's question does not get due attention just like it can happen on the main site?

 
0
Q: Is this integral convergent ??

hichamgaussI have this question is this integral converge always $$\int_{0}^{1}x^\alpha|ln(x)|^\beta dx$$ where $$\alpha >0 ,, \beta >0$$ thanks

Title contains ??. Short question.
0
Q: Help in discrete math

Cyber OperationsWhat do you obtain when you apply the projection P2,3,5 to the 5-tuple (a, b, e, d, e)?

Welcome to Math.SE, Cyber Operations. Words such as help do not add information to titles. Please edit the title so that it better describes the specifics of your question. Do not hesitate to make it longer or include a formula if needed. More tips at: How to ask a good question? (autocomment)Normal Human 21 secs ago
0
Q: What is the particular solution of this second order differential equation?

p.gurungHow do I solve this second order PDE? $\frac{\partial^2 X}{\partial t^2}+X= \cos(\omega t)+\frac{1}{4}(1-exp(-\tau +F(\sigma))^\frac{-3}{2} [\sin(6t-6\psi)+3\sin(4t-4\psi)+3\sin(2t-2\psi)]$ $\omega$ is a constant. $F(\sigma)$ is just a general function of $\sigma$. $\tau$ and $\sigma$ are slow...

Tagged pde, differential-equations.
0
Q: Solving a trigonometric polynomial equation.

Ag Ncan anyone please explain how we can find number of distinct solutions of the following equation : (5/4)cos^2(2x) + cos^4(x) + sin^4(x) + cos^6(x) + sin^6(x) = 2 between the interval [0,2pi] thanks for your answers in advance

Welcome to Math.SE, Ag N. This site uses MathJax formatting of formulas. More tips at: How to ask a good question? (autocomment)Normal Human 21 secs ago
 
1:34 PM
-3
Q: How To Upload 360 In Facebook With perfect Setting

Danyial Shahid AliFacebook By Default Set My Video To This Setting And Preview IS Very bad Actually I Want This But I Don't Know How To Set Setting Of This **The Video IS Look Like This And Link Is View **

-1
Q: Delete protection for questions with a new answer

bolovCurrent situation A questions cannot be deleted by their user if they have an upvoted/accepted answer or multiple answers. The motivation with which I agree is that questions with good answers are valuable to the community. Problem However this leaves the following scenario open (which has ha...

 
0
Q: Is there a convex polygon such that it cannot be tiled with some number of congruent connected pieces?

Ante PaladinSo the title says it all. I assume that polygons have straight line segments as their edges and that they have finite number of edges. The number $n$ of pieces is, of course, $n>1$, to avoid triviality that every polygon tiles itself.

Questions tend to get more attention when they have a tag for a broad area of mathematics relevant to the question. Some of these tags might fit. (autocomment)Normal Human 25 secs ago
0
Q: A question about the kernel of a Frobenius group

A-213A Frobenius group $G$ is a transitive permutation group which is not regular, but in which only the identity has more than one fixed point. Let $K=\{x\in G \vert x=1 \ or \ x \ has \ no \ fixed \ point \}$. Then $K$ is called the Frobenius kernel of $G$. It is known that if $G$ is finite, the...

Words such as question do not add information to titles. Please edit the title so that it better describes the specifics of your question. Do not hesitate to make it longer or include a formula if needed. More tips at: How to ask a good question? (autocomment)Normal Human 24 secs ago
0
Q: question on right angle triangle

Sarnavo SarkarLet ABC and DBC be two equilateral triangle on the same base BC,a point P is taken on the circle with centre D,radius BD. Show that PA,PB,PC are the sides of a right triangle.

Words such as question do not add information to titles. Please edit the title so that it better describes the specifics of your question. Do not hesitate to make it longer or include a formula if needed. More tips at: How to ask a good question? (autocomment)Normal Human 21 secs ago
0
Q: Help in discrete math for "Relations and Their Properties"

SOOTLet S be a set with n elements and let a and b be distinct elements of S. How many relations are there on S such that: a) (a, b) ∈ S? b) (a, b) ∉ S? c) there are no ordered pairs in the relation that have “a” as their first element? d) there is at least on ordered pair in the relation that has “...

Title contains help.
0
Q: entire function find the growth order of f+g,fg

mathdingdongLet f,g  be non-zero entire functions having growth orders o(f),o(g)   respectively. (a) Find the growth of order of f+g  . (b) Find the growth of order of fg.

Short question.
0
Q: Problem with sign in solving equation of a line with slope and one point

nulliusinverbaI am trying to solve a question where you are given two points, (2, 0) and (3, 5), asked to find the equation - seems simple enough, I get $y = 5x - 10$, which my plotting on a graph seems to confirm is correct. Then, given the point (-1, -1), it asks to find the equation of the perpendicular li...

Title contains problem.
 
2:33 PM
0
Q: conditional probability question - expect value

sheronI got a bit confused in the below question of probability. Please help me to clarify...thanks! May will read either one chapter of history book or one chapter of comics. If the number of misprints in a chapter of history book and comics are Poisson distributed with mean 2 and 5 respectively, t...

Words such as question do not add information to titles. Please edit the title so that it better describes the specifics of your question. Do not hesitate to make it longer or include a formula if needed. More tips at: How to ask a good question? (autocomment)Normal Human 21 secs ago
0
Q: How would you calculate cos(2arcsin(4/5)) without calculator

CoolKidWithout the 2, the answer is easily 3/5. But with 2, how do you visualize the new triangle in order to get the result?

Short question.
0
Q: Binomial Distribution using probability

Ipsita I came across a binomial distribution question which states that X follows B(20, 0.75). Is it possible to find the median with only this much information?

Consider adding a tag for a broader subject area to which the question belongs. Some of these tags might fit. (autocomment)Normal Human 26 secs ago
0
Q: Prove the solution to PDE problem is the trivial

BCLC$$u_t = c^2u_{xx} \mid (x,t) \in (0,L) \times (0, \infty)$$ $$u_x(0,t) = u_x(L,t) = 0 \mid t > 0$$ $$u(x,0) = 0 \mid x \in [0,L]$$ I tried energy method based on Pinchover and Rubinstein (*): $$\text{Let } E(t) = \frac{1}{2} \int_{0}^{L} u^2 dx$$ $$\to E'(t) = \frac{1}{2} \int_{0}^{L} 2uu_t ...

Title contains problem.
0
Q: In the weak formulation of the Poisson equation, why is the boundary condition included in the integration of the weighted residual?

halayangubeIn the weak formulation of the Poisson equation $\nabla^2u = g$ with boundary conditions $u = \bar{u}$ on $\Gamma_e$ and $\frac{\partial{u}}{\partial{n}} = \bar{q}$ on $\Gamma_n$, why is the integration of the weighted residual expressed as $$I = \int_\Omega w(\nabla^2u-g) d\Omega - \int_{\Gamma...

Welcome to Math.SE, halayangube. Questions tend to get more attention when they have a tag for a broad area of mathematics relevant to the question. Some of these tags might fit. (autocomment)Normal Human 21 secs ago
0
Q: Approximation by sin waves using DFT on python. What's wrong?

faceless wandererHell! I'm writing the prorgram on python that can approximate time series by sin waves. The program uses DFT to find sin waves, after that it chooses sin waves with biggest amplitudes. Here's my code: __author__ = 'FATVVS' import math # Wave - (amplitude,frequency,phase) # This class was cre...

0
Q: Monty Hall Problem - Why is the conditional probability not considered?

WolffSo we all know the Monty Hall problem where there's you're on a gameshow and you have 3 doors, one of which has a car behind it and the other 2 a goat. I understand that if you pick one door, there is a 1/3 chance of winning and that the other two doors combined have a 2/3 chance of having a car...

Title contains problem.
 
3:00 PM
0
Q: What does marked as duplicate by Community♦ mean?

kasperdI came across a question which said marked as duplicate by Community♦ 1 hour ago Usually when a question is closed there will be an indication of who closed it - either five users or one moderator. This one appears to have been closed without going through review, and the question has no vo...

 
3:17 PM
0
Q: How to tackle a Fourth order ODE Euler-Bernoulli equation: d^4y(x)/dx^4 = f(x)/EI

Adam MacmartinI asked this earlier but since have been asked to be more specific. I'm trying to find the curve of a cantilever beam y(x), which is described by a fourth order Euler-Bernoulli beam equation: d^4y(x)/dx^4 = f(x)/EI. f(x) is the force per unit length (uniformly distributed load), E is Youngs m...

Welcome to Math.SE, Adam Macmartin. This site uses MathJax formatting of formulas. More tips at: How to ask a good question? (autocomment)Normal Human 21 secs ago
0
Q: A basic doubt about the definition of Convergence of Series

Shrey AryanThe definition of convergence states that the nth Partial Sum converges. Suppose the Sn (Partial Sum) converges to 0. Will that be considered as convergence or not?

Words such as doubt do not add information to titles. Please edit the title so that it better describes the specifics of your question. Do not hesitate to make it longer or include a formula if needed. More tips at: How to ask a good question? (autocomment)Normal Human 21 secs ago
Short question.
0
Q: How to substitute x into binomial

Jake Write down the first three terms in the binomial expansion of (1 – x)^15. By substituting x = 0.01, find an approximate value for 0.99^15 I understand the first part however how do I substitute x? The first three values are: -x^15 + 15x^14 - 105x^13

Welcome to Math.SE, Jake. This site uses MathJax formatting of formulas. Questions tend to get more attention when they have a tag for a broad area of mathematics relevant to the question. Some of these tags might fit. More tips at: How to ask a good question? (autocomment)Normal Human 22 secs ago
0
Q: Proof regarding the convergent series of a subsequence

ArthurGiven a null sequence $(a_n)_{n\in\mathbb{N}}$ and $a_n\in\mathbb{R}^{>0},\forall n\in\mathbb{N}$ I need to prove that $\forall\epsilon >0,\exists (a_{n_k})_{k\in\mathbb{N}}$ such that $(\sum_{k=1}^{\infty}a_{n_k})<\epsilon$. So basically, I think I need to construct a (geometric?) series whic...

Tagged proof-writing.
0
Q: Question concerning umbilic torus topology

JoeIn the below two images you see one with three lines leaving point A each 120 degrees apart and a umbilic torus made from twisting a triangle around a circle. I have tried to construct an umbilic torus with, rather than a triangle, the shape below. I am doing this with the intent of investigati...

Words such as question do not add information to titles. Please edit the title so that it better describes the specifics of your question. Do not hesitate to make it longer or include a formula if needed. More tips at: How to ask a good question? (autocomment)Normal Human 23 secs ago
0
Q: question on polynomial

Sarnavo Sarkar$f(x)$ is a fifth degree polynomial. It is given that $f(x)+1$ is divisible by $(x-1)^3$ and $f(x)-1$ is divisible by $(x+1)^3$. Find $f(x)$.

Words such as question do not add information to titles. Please edit the title so that it better describes the specifics of your question. Do not hesitate to make it longer or include a formula if needed. More tips at: How to ask a good question? (autocomment)Normal Human 21 secs ago
 
4:09 PM
0
Q: Set Complex symbol

AraI have a rather trivial question, when the book states: "Assume that the function of n variables f is of class $\mathsf{C^2} $" I know that $\mathsf{C}$ is the complex set, but how about squared? What can I conclude from that. Sorry for my silly question.

Tag (notation) should not be the only tag a question has. Please add a tag for a subject area to which the question belongs. (autocomment)Normal Human 26 secs ago
0
Q: Does $f'(x) → 3$ as $x → 0$ imply $f'(0)$ exists? (Counterexample verification)

zagadka314If $f(x)$ is a real function such that $f'(x) → 3$ as $x → 0$, does that mean $f'(0)$ exists? I ask this because of problem 9 on page 115 of Rudin's Principles of Mathematical Analysis. The actual question is slightly different: Let $f$ be a continuous real function on $ℝ^1$, of which it is k...

Tagged proof-verification.
 
4:26 PM
0
Q: Tag description problem

MaloubobolaIt seems that there is a bug with the tag description (here is the one from java) When I switch the tab (from java to android for example), the description block does not change. See this image, I am on the android tab, but I still have the java description (so the links redirects to java page...

 
0
Q: question about oscillatory integral?

MuniainToday I study about oscillatory integral: let $q$ be a nondegenerate real quadratic form on $\mathbb{R}^n$, $a\in A^m$ and $\psi\in C_0^\infty$ such that $\psi=1$ if $|x|\leq 1$ and $\psi=0$ if $|x|\geq 2$, so we have $$\lim_{n\to \infty} \int{e^{i q(x)} a(x) \psi(2^{-n} x)}dx$$ exist, any by def...

0
Q: limit of multivariable functions plus a difficult problem in continuity

karhasAt my multivariable calculus class we gave this definition for the limit of a function: Definition: Let $ \mathbb{R}^n \supset A $ be a open set , let $f:A \to\mathbb{R}^m $ be a function, let ${\bf x_0}$ be a point of $A$ and ${\bf P}$ a point of $\mathbb{R}^m$. To say that $f$ ...

Title contains difficult, problem.
Words such as question do not add information to titles. Please edit the title so that it better describes the specifics of your question. Do not hesitate to make it longer or include a formula if needed. Consider adding a tag for a broader subject area to which the question belongs. Some of these tags might fit. More tips at: How to ask a good question? (autocomment)Normal Human 21 secs ago
0
Q: Poisson Distribution using probabilities

Ipsita If X follows a poisson distribution where P(X = 1) = 0.4, P(X = 2) = 0.6, what is the value of P(X = 0)? How will I solve this question?

Short question.
0
Q: How to prove that $\frac{1}{3}e^{2t} + \frac{2}{3}e^{-t}\leq e^{2t^2}$

user34870How to prove that, for every real $t$, one has $$\frac{1}{3}e^{2t} + \frac{2}{3}e^{-t}\leq e^{2t^2}?$$

Short question.
0
Q: How many coefficients do you need to determine a modular form

A.champolionHere is the question i'm particularly interested in : Let $f$ be a modular form, suppose we know the $a_p(f)$ for all but finitely many prime $p$. Is this enough to know the modular forms i.e. to know all the $a_n$ ? If it is true can you give a proof or reference ? I'm also interested (but it'...

Questions tend to get more attention when they have a tag for a broad area of mathematics relevant to the question. Some of these tags might fit. (autocomment)Normal Human 32 secs ago
0
Q: Finding Taylor/Laurent series for complex functions? - confused about general method

KamilI'm confused about the correct method for finding Taylor/Laurent series for complex functions. For example, I'm given this function $$ f(z) = \frac{z^2-1}{(z+2)(z+3)} $$ and I need to find the Taylor/Laurent series centered at $0$ for all $z \in \mathbb{C}, |z| \neq 2, 3$. From my understanding, ...

Title contains confus.
0
Q: Sum of two squares en equality

sriramIf a^2 + b^2 not equal to c^2 +d^2 and if a not equal to b and c not equal to d and if a b c d are greater than or equal to 0 AND NOW IF L GREATER THAN OR EQUAL TO 1 THEN TO PROVE (a+L)^2 + (b +L)^2 not equal to (c + L)^2 + (d +L)^2

0
Q: Values of a function with a rational domain continuous over an interval

GeorgeIf $f(x)$ is continuous over [0,1] and $f$ only takes rational values, if $f( {1 \over 2})={1\over2}$, how do I prove $f(x)={1\over2}$ everywhere on [0,1]?

Welcome to Math.SE, George. Consider adding a tag for a broader subject area to which the question belongs. Some of these tags might fit. (autocomment)Normal Human 31 secs ago
0
Q: proving that $\sqrt[n]{b_n}\to 1$

dinaGiven $(a_n)_{N\in\mathbb{N}}$ such that $a_n \geq 0$, and $a_n\to a \geq 0$. how can i prove that $\sqrt[n]{b_n}\to 1$ when $n \to \infty$?

Short question.
0
Q: Confusion regarding The Fundamental Theorem of Calculus

xasthorI'm currently studying Calculus from Stewart's book, and for the The Fundamental Theorem of Calculus Pt. 1, he defined a function $g(x) = \int_0^x f(t) dt$ which represented the area under $f(x)$ from $0$ up to $x$ and proved that $g(x)$ is the antiderivative of $f(x)$ and, in this case, if I plu...

Title contains confus.
0
Q: Structure of dense subset of $\mathbb{R}$

RFZHere user Mario Corneiro formulated statement which I find really interesting and useful and I proved it and I would like to know is my proof correct? If sequence $\{a_n\}_{n=1}^{\infty}$ implies the following conditions: 1) $a_1<a_2<\dots<a_{n}<\dots$ 2) $a_n\to \infty$ as $n\to...

Tagged proof-verification.
0
Q: Projection of vector on $(x_1,x_3)$-space

M. MeyerIf I have a three-dim. vector $$ (x_1,x_2,x_3)\in\mathbb{R}^3, $$ what then is its projection onto the $(x_1,x_3)$-space? I am not sure.

Short question.
0
Q: Linear Independence of Polynomials

DanI'm trying to show that {1, x, x^2,...,x^n} is a linearly independent set (in P_n) without being circular; so without using either the Fundamental Theorem of Algebra or the fact that this is the standard basis for P_n. I understand if a,b,c are all distinct positive integers and t1, t2 arbitrar...

Welcome to Math.SE, Dan. This site uses MathJax formatting of formulas. More tips at: How to ask a good question? (autocomment)Normal Human 29 secs ago
0
Q: Real Analysis Help

Math GuyFor #1 I am able to prove it but I am not sure what kind of example can be used and how I would prove that it would fail. For #2, I am not sure how to go about it. 1) Let {$A_n$} be an infinite sequence of measurable sets increasing to A. Prove that m(A)=$\lim_{n\to \infty}$ m($A_n$). Give an ...

Words such as help do not add information to titles. Please edit the title so that it better describes the specifics of your question. Do not hesitate to make it longer or include a formula if needed. More tips at: How to ask a good question? (autocomment)Normal Human 22 secs ago
0
Q: prove that $\lim\limits_{n\to\infty}\frac{a_{n+1}}{a_n}=1$

dinaGiven $\lim_{n\to\infty} a_n = a \neq0$. Need to prove that $\lim\limits_{n\to\infty}\frac{a_{n+1}}{a_n}=1$. So i know that if $\lim_{n\to\infty} a_n = a$ so $\lim_{n\to\infty} a_{n+1} = a$ for the getting $1$, but how do i prove it?

Title contains \limits.
0
Q: Find all integer solutions to 4x congruent to 13 (mod 3).

Jacobi am confused on how to go about solving for this. perhaps i am not understanding how to solve congruence problems. any help will be appreciated.

Short question.
0
Q: Proof with mathematical induction.

ParketPlease help to prove this, using mathematical induction: $$(x_1 + x_2 + ... +x_n)(\frac{1}{x_1} + \frac1{x_2} + ... \frac{1}{x_n}) \ge n^2, $$ $$ x_i > 0, $$ $$i = 1, 2, ... n$$

Short question.
 
5:43 PM
0
Q: Discrete Mathematics | Difference and Shift Operators

Vitiello$\sum_{k=1}^n k2^k = 2^{n+1}(n-1)+2$ How can I justify that using difference and shift operators?

 
0
Q: best way to edit a question where OP asked more than one question?

Ben BolkerIn this question the OP is asking two separate questions. The original version asked only a single question (which is actually a duplicate, but it took me a little while to find it); after getting an answer to that question, they added a new question. Now the question has reasonable answers to b...

 
Short question.
0
Q: bijection form Z into (0,1)^Q

sumudu madushanFind a bijection form Z into (0,1) ^ Q can anyone help me with this

Short question.
 
6:06 PM
0
Q: In what way am I troll?

BCLCDid has accused me of being a troll. I don't see how. Did was being unclear as to what he meant by 'first implication'. I don't know if he means first in my answer to my question, first in my question, first after the '?', first after the ':', etc. Instead he calls me a troll for not understandin...

0
Q: Incorrectly failed first post review audit because of a *positive* comment on a good answer

plamutWhile reviewing the First Posts queue I was presented this question. It seemed like a very good question for which I wanted to compliment the first poster. I added a positive comment that also contained a hint to an answer, i.e. a link to the Wikipedia's article on the halting problem and briefl...

 
0
Q: (online) Tool to calculate $E(\mathbb{Q})/2E(\mathbb{Q})$ for $E: y^2 = x(x^2 + 3x + 5)$

KrijnFor some exercise I need to compute the generators of $E(\mathbb{Q})/2E(\mathbb{Q})$, where $E: y^2 = x(x^2 + 3x + 5)$ I did this by the algorithm from Cassels book 'Lectures on Elliptic Curves' and got the answer $< (0,0), (1,3) >$ which seems reasonable. A friend of mine did the same computati...

Consider adding a tag for a broader subject area to which the question belongs. Some of these tags might fit. (autocomment)Normal Human 29 secs ago
0
Q: Is my proof of $\lim _{ x\rightarrow \infty }{ \frac { x+8 }{ x+3 } =1 } $ correct?

Cherry_Developer$$\lim _{ x\rightarrow \infty }{ \frac { x+8 }{ x+3 } =1 } $$ Proof: Let $\epsilon > 0$ $$\left| \frac { x+8 }{ x+3 } -1 \right| <\epsilon $$ $$\Longrightarrow \left| \frac { x+8 }{ x+3 } -\frac { x+3 }{ x+3 } \right| <\epsilon $$ $$\Longrightarrow \left| \frac { 5 }{ x+3 } \right| <\epsil...

Tagged proof-verification.
0
Q: Existence and Uniqueness Problem

A. ÇalışırConsider IVP = ln(t) dy/dt +2y = tan(t) and y(1.5) = -2 . Find the longest open interval where the unique solution to IVP is certain to exist.Can you help me out ?

Welcome to Math.SE, A. Çalışır. Consider adding a tag for a broader subject area to which the question belongs. Some of these tags might fit. (autocomment)Normal Human 26 secs ago
0
Q: Let $R$ be a ring without unity. Suppose that $char R=n$. Is there an element $r\in R$ with $|r|=n$?

bfhahaLet $R$ be a ring without unity (not necessarily commutative). Suppose that $char R=n$. Does there must exists an element $r\in R$ with $|r|=n$?

Short question.
 
0
Q: Tag [Bootstrap-modal] return no result or connection reset

SheharyFound this issue with following link http://stackoverflow.com/?tab=recommended&subtab=recent&hideReview=on&minScore=0 When filter by tag: Bootstrap-modal, takes too much time to search and return's Oops! something bad Happened OR The connection was reset Checked on 2 different laptops w...

 
0
Q: Find all integer solutions to 5x≡3mod3

Jacobthis is what i got but im not sure if it is correct. 5x≡3mod3 3*5x≡3*mod3 x≡15x≡6mod3 x≡0mod3 x=3k k belongs to integers

Short question.
0
Q: Solving a difficult differential equation

TdonutHere's an interesting differential equation: $$f''(x) = \frac{x(f'(x))^2+f(x)^2}{f(x+1)}$$ It was created by a someone I know, and I am wondering if anyone can solve it. I mainly want to see what other people do on it before I reveal my own work.

Words such as difficult do not add information to titles. Please edit the title so that it better describes the specifics of your question. Do not hesitate to make it longer or include a formula if needed. More tips at: How to ask a good question? (autocomment)Normal Human 27 secs ago
0
Q: Forward difference operator

VitielloWhat does $\Delta^{-1}$ mean? I have seen it in a question such as "justify that $\Delta^{-1}k^{(n)} = {k^{n+1}\over{n+1}}$". Thanks for your help.

Short question.
 
6:36 PM
0
Q: in set theory what is the result of {{A},{B}}\{{A}}?

NivThe original question is: given A={0,1} B = {{0,1},{1}} What is the result of P(B{A})

Short question.
0
Q: Question about positive matrix

ThamírisIf I have a nonsingular matrix M, How can I prove that M*M is positive definite? M* is the transpose matrix of M. Thank You!!

0
Q: Modular arithmetic problem splited

user2254798i have 3x mod 10=1 and i split to 3x mod 5 = 1 3x mod 2 = 1 now how to solve that?

Title contains problem. Short question.
Words such as question do not add information to titles. Please edit the title so that it better describes the specifics of your question. Do not hesitate to make it longer or include a formula if needed. More tips at: How to ask a good question? (autocomment)Normal Human 41 secs ago
0
Q: Linear Independence for Indeterminates

sobrio35I am trying to prove that {1, x, x^2, ..., x^n} is a linearly independent set with no hand waving and without using the fact that it is a basis for P_n or the Fundamental Theorem of Algebra. Suppose (a_0) + (a_1)x + ... + (a_n)x^n = 0. How do I show a_i = 0 for 1 <= i <= n?

Welcome to Math.SE, sobrio35. This site uses MathJax formatting of formulas. More tips at: How to ask a good question? (autocomment)Normal Human 27 secs ago
0
Q: Help integrating a function

kRiShNa_96$\frac{2}{\pi}\int_{0}^{\pi}\cos(\frac{x}{2}).\cos(nx) dx$ please show step by step solution

Words such as help do not add information to titles. Please edit the title so that it better describes the specifics of your question. Do not hesitate to make it longer or include a formula if needed. More tips at: How to ask a good question? (autocomment)Normal Human 25 secs ago
0
Q: If $p$ is prime and $\gcd(m,p) = 1$ show that $\gcd(m,p^k) = 1$

dableIf $p$ is prime and $\gcd(m,p) = 1$ show that $\gcd(m,p^k) = 1$ where $k\geq1 $. I think I have come up with a solution: Suppose $m$ has prime factorization $m = p_1^{a_1}...p_n^{a_n}$ where $p_i$ is prime. Since $\gcd(m,p) = 1$ none of $p_i = p$. Now if $\gcd(m,p^k) \neq 1$ then the $\gcd$ mus...

Tagged proof-verification.
0
Q: How to generating Shadowed Rician random number in MATLAB

MeetI'm required to generate random number which follows Shadowed Rician distribution as enter image description here

Short question.
0
Q: Trig arctan Equation Help

user3469157I have a field of view on something I am working on of 41.5933 (is correct) however the equation that was supplied to me that was supposedly used to calculate it for some reason does not add up to the 41.5933 I get 4.07958271 when I try it in unless I am not doing it correct. Any help to determin...

0
Q: Identifying 2 poisoned wines out of 2^n wines

blackpenThis problem has been asked and discussed in following posts: Logic problem: Identifying poisoned wines out of a sample, minimizing test subjects with constraints, Finding 2 poisoned bottles of wine out of a 1000 Identifying poisoned wines I know that there are better optimized solutions to...

Words such as help do not add information to titles. Please edit the title so that it better describes the specifics of your question. Do not hesitate to make it longer or include a formula if needed. This site uses MathJax formatting of formulas. More tips at: How to ask a good question? (autocomment)Normal Human 20 secs ago
0
Q: $Lim \coprod F_{k}\cong \prod Lim F_{k}$

LiddoI have some doubts with this problem because The concept of Limit is confusing for me: "If one category $J$ is a disjoint union (coproduct) $\coprod_{k}F_{k}$ of categories $J_{k}$ for index $k$ in some set $K$, with $I_{k}:J_{k}\rightarrow J$ the injection of the coproduct, then each functor $...

A title should not be all-MathJax; having some plain text helps with search and navigation. (autocomment)Normal Human 29 secs ago
0
Q: Double Dual Spaces linear maps of linear maps confusing

Arcane1729Hi I am stuck on a problem about dual spaces which I've spent hours on but I just cant grasp the idea of functions of functions- The problem in mind uses the vector space V of polynomials of degree $\leq 2$ over the field $\mathbb{R}$ and I have been given 3 linear maps from V to R- e.g a map $\p...

Title contains confus.
0
Q: Is sine related to circle or curve?

KartikAs tangent is a line passing through a curve from one point secant from two points so is sine also something like this?

Short question.
0
Q: Marginal and conditional distribution for a continuous and a discrete variables

MousaConsider a one-dimensional classification problem with X = R and Y = {-1, +1}: Visualize the marginal distribution p(x) and the conditional distributions p(y = -􀀀1 | x) and p(y = +1 | x). Guess from the visualization of p(y = 􀀀-1 | x) and p(y = +1 | x) what the Bayes-optimal classifier is li...

0
Q: Base cases in proof by induction

P999How can I use base case of n=1 to prove the following? $$\forall z \in Z: \forall n \in N: n \gt 0 \Rightarrow \lceil z/n \rceil = \lfloor (z+n-1)/n \rfloor.$$ By assuming: $$\forall l \in Z: \forall m \in N: m \gt 0 \Rightarrow [\exists q \in Z: \exists n \in N: l = qm+n \wedge n < m]$$ My wor...

Tagged proof-writing.
0
Q: Finite topological space

user290064Let $X$ be a Hausdorff topological space such that every closed subset has finitely many connected component. How can I verify that $X$ is finite?

Short question.
0
Q: integrations by parts help

AlbertWolfgangShow $\int_0^\infty 2y^2e^{-y^2} dy = .886$ $2\int_0^\infty y^2e^{-y^2} dy$ Then by parts; $f=y^2$ $dg=e^{-y^2}$ $df=2y$ $g=-2ye^{-y^2}$ $2\int_0^\infty y^2e^{-y^2} dy$ = $-2y^3e^{-y^2} -\int_0^\infty -4y^2e^{-y^2}$=$-2y^3e^{-y^2} +4\int_0^\infty y^2e^{-y^2}$ equivalently; $-2\int_...

Words such as help do not add information to titles. Please edit the title so that it better describes the specifics of your question. Do not hesitate to make it longer or include a formula if needed. More tips at: How to ask a good question? (autocomment)Normal Human 26 secs ago
0
Q: How to create a random matrix whose spectral radius $<1$

RoyGiven a matrix size $n$, I want to produce a matrix $A$ with $\rho(A)<1$, which is not diagonal. Is there a way to do so? Thanks.

Short question.
0
Q: Square of Elementary Matrix Proof

ViennaI'm having trouble proving the following statement: "There exists an elementary matrix E_1 such that (E_1)^2 = I" I'm thinking about how the inverse of E_1 is qua; to E_1 (E_1^-1 = E_1) but I'm not sure how to show the product of it, if that is even the right step towards the proof. Could some...

0
Q: Consider the curve given by y^2=2+xy

John Yajoo(A) Show that dy/dx=y/2y-x, (B) Find all points (x,y) on the curve where the line tangent has a slope of 1/2, (C) Show that there are no points (x,y) on the curve where the line tangent to the curve is horizontal, (D) Let x and y be functions of time t that are related by the equation y^2=2+xy. A...

Body punctuation.
0
Q: Rosenlicht's "Introduction to Analysis" solution manual?

luka5zDo you know if there is some kind of solutions manual to Rosenlicht's "Introduction to Analysis"? I would be very grateful for a link (if there is one). Thank you.

Short question.
0
Q: number line basic theoretical question (courant calculus)

user122415 The text is from courant calculus volume 1. First few pages. So obviously I don't understand this part. It says we divide each interval on the number line by q parts. Thus each subdivision length will be 1/q. I understand this. Then it says every point P is p/q, or lies between p/q and ...

0
Q: A birthday problem

alex alexeqAmong 5 people, what is the probability that exactly 2 of them are born in the same month.

Title contains problem. Short question.
Words such as question do not add information to titles. Please edit the title so that it better describes the specifics of your question. Do not hesitate to make it longer or include a formula if needed. This site uses MathJax formatting of formulas. More tips at: How to ask a good question? (autocomment)Normal Human 21 secs ago
0
Q: Independence of Multiple Events

j.statIf the events A and B are independent, and the events A and C are independent, is it true that the events A and B − C are also independent? Prove, or give a counterexample.

Questions tend to get more attention when they have a tag for a broad area of mathematics relevant to the question. Some of these tags might fit. (autocomment)Normal Human 23 secs ago
 
7:31 PM
0
Q: Changing filter criteria in the new nav should send the user back to page 1

Santa ClausIf I change filter criteria in the new nav (ie. adding a tag to filter by), it sends me to the same page # of the new search criteria as the page I was on before. Start by looking at page 26 of the bounties list: http://stackoverflow.com/questions/bounties/ending?page=15 Then, try to a...

 
0
Q: Linear Transformations, Linear Algebra

Laughing HorseLet T:P3→P3 be the linear transformation such that T(−2x^2)= −2x^2 − 2x, T(0.5x + 2)=3x^2 + 4x−2, and T(2x^2 − 1)= 2x + 1. Find T(1), T(x), T(x2), and T(ax^2 + bx + c), where a, b, and c are arbitrary real numbers. I understand how to find T(x^2) where you just divide the given T(-2x^2) by -2 t...

0
Q: Is there an easy way to graph tupper's self referential formula?

BanbadleIs there an easy way to graph tuppers self referential formula between $$k<y<k+17 $$for a value of K? Like any sites or programs? or would i have to do it the long way of dividing by 17, turn it into binary, and draw it out myself?

Title contains easy. Short question.
Welcome to Math.SE, Laughing Horse. This site uses MathJax formatting of formulas. More tips at: How to ask a good question? (autocomment)Normal Human 1 min ago
0
Q: How can I find the order of $\overline {15}+ <\overline 4>$ in $\Bbb Z_{45}/<\overline 4>$?

memontofind the order of $\overline {15}+ <\overline 4>$ in $\Bbb Z_{45}/<\overline 4>$ how can I start? is it $\Bbb Z_{45}/<\overline 4>\cong \Bbb Z$

Short question.
0
Q: Time variant rotation matrix

user290069I have the differential equation dR(t)/dt=S(t)R(t), I know that S is a skew symmetric matrix and that R(0) is a rotation matrix. How do I prove that R is then a rotation matrix at any time t?

Short question.
 
7:50 PM
0
Q: Distribution Function of Binomial Random Variable

Jayson HumarangLet X be an rv with pmf $p_k= (nCk) p^k (1-p)^(n-k)$ [binomial]. If F is the corresponding df, find the distribution of F(X). I know for certain that I need to get the sum of k binomial rv's, however I'm having a hard time in doing it. I read from books that it will be needing of mathematical in...

Consider adding a tag for a broader subject area to which the question belongs. Some of these tags might fit. (autocomment)Normal Human 28 secs ago
 
3
Q: Are answers that contain only commented code acceptable?

bobI found this answer today: The code itself is fine, but there's nothing explaining the problem or why the provided code fixes the problem. I commented on the answer requesting the user to add more details: Please explain your code, instead of doing all the work for the OP. The user then...

1
Q: Add all other sites to the Flag > Closing > Offtopic > Migration list

try-catch-finallyCurrently the dialog only contains the sites Stackexchange Meta superuser.com tex dba stats This is an odd collection of sites. Most time I guide people to Unix & Linux Serverfault Wordpress, Magento etc. Programming Code Review (For Stackoverflow, at least these should be available.) ...

 
0
Q: T is scalar multiple of identity operator.

Anurag JainLet T be a linear operator on a n-dimensional vector space V.Suppose that T commutes with every diagonalizable linear operator on V .Prove that T is a scalar multiple of identity operator.

Short question.
0
Q: Path lifting theorem. Correctness of the proof.

MihailLet $p:X\to B$ be a covering and $g:I\to B$ be a path on $B$ with $p(x_0)=g(0)=b_0$. Then there exists unique lifting path $f:I\to X$ s.t. $g=pf$ and $f(0)=x_0$. I didn't get fully the proof from the book, so I've tried to prove it by myself, but I'm not sure about its correctness. Proof: Since ...

Tagged proof-verification.
 
00:00 - 20:0020:00 - 00:00

« first day (12 days earlier)      last day (562 days later) »